Olá, Comunidade!

Vocês devem ter notado que o site ficou um período fora do ar (do dia 26 até o dia 30 de maio de 2024).

Consegui recuperar tudo, e ainda fiz um UPGRADE no servidor! Agora estamos em um servidor dedicado no BRASIL!
Isso vai fazer com que o acesso fique mais rápido (espero 🙏)

Já arrumei os principais bugs que aparecem em uma atualização!
Mas, se você encontrar alguma coisa diferente, que não funciona direito, me envie uma MP avisando que eu arranjo um tempo pra arrumar!

Vamos crescer essa comunidade juntos 🥰

Grande abraço a todos,
Prof. Caju

DemonstraçõesTécnica olímpica - Lifting the exponent (LTE)

Fórum de coletânea das melhores demonstrações de teoremas de matemática.
Se você quiser postar uma demonstração aqui, poste, inicialmente, no fórum correspondente utilizando o título "Demonstração Teorema X" e substitua com o nome do teorema/fórmula que você postou e, depois, envie o link para um moderador pedindo para sua mensagem ser movida para o fórum "Demonstrações". Somente moderadores poderão mover sua mensagem para este fórum.

Moderador: [ Moderadores TTB ]

Avatar do usuário

Autor do Tópico
Ittalo25
5 - Mestre
Mensagens: 2349
Registrado em: 18 Nov 2013, 22:11
Última visita: 27-03-24
Agradeceu: 299 vezes
Agradeceram: 1401 vezes
Jan 2021 12 22:15

Técnica olímpica - Lifting the exponent (LTE)

Mensagem não lida por Ittalo25 »

Definição: Seja p um número primo. Se [tex3]e_p(a)=k [/tex3] , então [tex3]p^k \mid a[/tex3] mas [tex3]p^{k+1} \nmid a[/tex3] . Ou seja, k é a maior potência de p que divide a.

Propriedade 1: Se p é primo ímpar, [tex3]p \nmid a[/tex3] , [tex3]p \nmid b[/tex3] , mas [tex3]p|(a-b) [/tex3] , então: [tex3]e_p(a^n-b^n)=e_p(n)+e_p(a-b) [/tex3]
Demonstração:
Resposta

O caso base [tex3]n=1 [/tex3] é óbvio, supondo [tex3]n\geq 2[/tex3] e ser verdade para todo [tex3]k<n [/tex3] , precisamos mostrar que também vale para n.
1° caso: [tex3]p \nmid n[/tex3] .
Sendo [tex3]d = a-b [/tex3] , então:
[tex3]a^{n}-b^n = (b+d)^n-b^n = d\cdot \(nb^{n-1}+\sum_{i=2}^{n}\left(\frac{n}{i}\right)b^{n-i}d^{i-2}\)[/tex3]
Agora [tex3]p|d [/tex3] mas [tex3]p \nmid n [/tex3] e [tex3]p \nmid b^{n-1} [/tex3] , ou seja:
[tex3]e_p(a^n-b^n) = e_p(d) = e_p(a-b) [/tex3]
Mas [tex3]e_p(n)=0 [/tex3] por hipótese, assim:
[tex3]e_p(a^n-b^n) = e_p(a-b)+e_p(n) [/tex3]

2° caso: [tex3]p \mid n[/tex3] .
Seja: [tex3]n=mp [/tex3] , [tex3]d=a^m-b^m [/tex3] e [tex3]b^{m} = c[/tex3] , então:
[tex3]a^n-b^n = (a^m)^p-(b^m)^p = (d+c)^p-c^p = pdc^{p-1}+d^p+\sum_{i=2}^{n}\left(\frac{p-1}{i}\right)d^ic^{p-i} [/tex3]
Agora: Para todo [tex3]2\leq i\leq p-1[/tex3] , [tex3]p|\left(\frac{p}{i}\right)[/tex3] e também [tex3]d^2|d^i [/tex3] .
Como [tex3]p|a-b [/tex3] e [tex3]a-b|a^m-b^m [/tex3] , então [tex3]p|a^m-b^m=d [/tex3] , mas como p é ímpar [tex3](p\geq 3)[/tex3] , então [tex3]pd^2 \mid d^p [/tex3]
Sendo assim, existe um inteiro k tal que [tex3]d^p+\sum_{i=2}^{n}\left(\frac{p-1}{i}\right)d^ic^{p-i} =kpd^2[/tex3]
Portanto: [tex3]a^n-b^n =pdc^{p-1}+kpd^2 = pd \cdot (c^{p-1}+kd) [/tex3] . Como [tex3]p|d [/tex3] mas [tex3]p\nmid c [/tex3] , então: [tex3]p\nmid c^{p-1}+kd [/tex3] .
Chegamos em: [tex3]e_p(a^n-b^n) = e_p(pd) = 1+e_p(d) = 1+ e_p(a^m-b^m) [/tex3]
Mas [tex3]m = \frac{n}{p}<n [/tex3] e por hipótese de indução: [tex3]e_p(a^m-b^m) = e_p(a-b)+e_p(m) [/tex3] , usando isso:
[tex3]e_p(a^n-b^n) = 1+ e_p(a^m-b^m) = 1+e_p(m)+ e_p(a-b) = e_p(a-b)+e_p(n) [/tex3]
Propriedade 2: Se p é primo, n ímpar e [tex3]p \nmid a[/tex3] , [tex3]p \nmid b[/tex3] , mas [tex3]p|(a+b) [/tex3] e [tex3]a+b\neq 0[/tex3] , então: [tex3]e_p(a^n+b^n)=e_p(n)+e_p(a+b) [/tex3]
Demonstração:
Resposta

Se [tex3]p\neq 2[/tex3] então basta aplicar a propriedade 1 para os números [tex3]a [/tex3] e [tex3]-b [/tex3] .
Se [tex3]p=2 [/tex3] , então:
[tex3]a^n+b^n = (a+b) \cdot (a^{n-1}-a^{n-2}b+....-ab^{n-2}+b^{n-1}) [/tex3]
Mas como [tex3]2\nmid a [/tex3] e [tex3]2\nmid b [/tex3] , então a e b são ímpares.
O número: [tex3](a^{n-1}-a^{n-2}b+....-ab^{n-2}+b^{n-1}) [/tex3] é a soma de n números ímpares, ou seja, é um número ímpar.
Sendo assim:
[tex3]e_p(a^n+b^n) = e_p(a+b) + e_p(a^{n-1}-a^{n-2}b+....-ab^{n-2}+b^{n-1})=e_p(a+b)+0 = e_p(a+b)+e_p(n)[/tex3]
Exemplo de aplicação 1: Sejam n>1 e a>0 inteiros e p um primo ímpar tal que [tex3]a^{p}\equiv 1\mod(p^n)[/tex3] . Prove que [tex3]a\equiv 1 \mod(p^{n-1}) [/tex3] .
Solução:
Resposta

Se [tex3]p^{n}| a^p-1[/tex3] , então [tex3]p| a^p-1[/tex3] , ou seja: [tex3]a^p \equiv 1 \mod(p) [/tex3]
Mas pelo pequeno teorema de Fermat: [tex3]a^p \equiv a \mod(p) [/tex3]
Sendo assim: [tex3]a \equiv 1 \mod(p) [/tex3]
Então: [tex3]p\nmid a [/tex3] , [tex3]p\nmid 1 [/tex3] e [tex3]p|a-1 [/tex3] .
Usando o LTE:
[tex3]e_p(a^p-1) = e_p(a-1)+e_p(p) = e_p(a-1)+1 [/tex3]
Como [tex3]a^{p}\equiv 1\mod(p^n)[/tex3] , então: [tex3]e_p(a^p-1) \geq n [/tex3] .
Ou seja: [tex3]e_p(a-1)+1 \geq n\rightarrow \boxed{e_p(a-1) \geq n-1} [/tex3]
Sendo assim: [tex3]p^{n-1}|a-1 [/tex3] e está provado.
Exemplo de aplicação 2: (Irlanda - 1996) Seja p um número primo, a e n inteiros positivos. Se [tex3]2^p+3^p=a^n [/tex3] , prove que [tex3]n=1 [/tex3]
Solução:
Resposta

Para p=2: [tex3]2^2+3^2 = 13^1 [/tex3]
Para p=3: [tex3]2^3+3^3 = 5 \cdot 7 [/tex3]
Para p=5: [tex3]2^5+3^5 = 11 \cdot 5^2 [/tex3]
Então supondo [tex3]p\geq 7[/tex3]
Para p ímpar: [tex3]2^p+3^p = 5 \cdot (2^{p-1}-3\cdot 2^{p-2}+....2\cdot 3^{p-2}-3^{p-1}) [/tex3]
Ou seja: [tex3]5|a^n [/tex3] .
[tex3]5 \nmid 2 [/tex3] , [tex3]5 \nmid 3 [/tex3] e [tex3]5|2+3 [/tex3] . Usando LTE:
[tex3]e_5(a^n) = e_5(2^p+3^p) = e_5(2+3) + e_5(p) = \boxed {1}[/tex3]
Exemplo de aplicação 3: Se a e n são inteiros positivos e [tex3]n|2^n+1 [/tex3] , então prove que [tex3]n=3 [/tex3] ou [tex3]9|n [/tex3]
Solução:
Resposta

Obviamente n tem que ser ímpar para dividir [tex3]2^n+1 [/tex3] , sendo assim:
[tex3]2^n+1 = (2+1) \cdot (2^{n-1}-2^{n-2}+....-2+1) [/tex3] , ou seja, n é múltiplo de 3.
[tex3]3 \nmid 2 [/tex3] , [tex3]3\nmid 1 [/tex3] e [tex3]3\mid 2+1 [/tex3] , usando LTE:
[tex3]e_3(2^n+1) \geq e_3(n) [/tex3]
[tex3]e_3(2+1)+e_3(3) \geq e_3(n) [/tex3]
[tex3]1+1 \geq e_3(n) [/tex3]
[tex3]\boxed{2 \geq e_3(n)\geq 1} [/tex3]
Exemplo de aplicação 4: (República Tcheca - 1996) Encontre todos os pares de inteiros positivos (x,y) tais que [tex3]p^x-y^p=1 [/tex3] , onde p é um primo ímpar
Solução:
Resposta

Como p é ímpar então: [tex3]p^x = y^p+1 = (y+1) \cdot (y^{p-1}-y^{p-2}+....-y+1) [/tex3] , ou seja: [tex3]y+1|p^x [/tex3] .
Mas então [tex3]y+1 [/tex3] é uma potência de p. Obviamente [tex3]p\nmid y[/tex3] , do contrário teríamos [tex3]p|1 [/tex3] . Então usando LTE:
[tex3]e_p(y^p+1) = e_p(y+1)+e_p(p) [/tex3]
[tex3]e_p(y+1) = x-1 [/tex3]
Ou seja: [tex3]p^{x-1} | y+1[/tex3]
Sendo assim:
[tex3]p^{x-1}= y+1 [/tex3] ou [tex3]p^x=y+1 [/tex3] .
1° caso:
[tex3]p^{x-1} = y+1 [/tex3]
[tex3]y^p+1 = py+p [/tex3]
[tex3]y^p-y =(y+1)(p-1) [/tex3] . Como [tex3]mdc(y,y+1)=1 [/tex3] , então:
[tex3]y| p-1\rightarrow p\geq y+1[/tex3] . Mas [tex3]y+1 [/tex3] é uma potência de p, ou seja: [tex3]y+1=p \rightarrow x=2[/tex3] .
Ou seja:
[tex3]p^2 = y^p+1 = (p-1)^p + 1 [/tex3]
[tex3](p-1) \cdot (p+1) = (p-1)^p [/tex3]
[tex3]p+1 = (p-1)^{p-1} [/tex3]
Mas p-1 é par: [tex3]p-1=2k [/tex3]
[tex3]p = (p-1-1)^{k}\cdot (p-1+1)^k [/tex3]
Então [tex3]k=1\rightarrow p=3[/tex3]
Solução: [tex3](x,y,p) = (2,2,3) [/tex3]
2° caso:
[tex3]p^{x} = y+1 [/tex3]
[tex3]y^p+1 = y+1 [/tex3]
[tex3]y^p = y [/tex3]
Único jeito seria [tex3]y=1 [/tex3] , mas daria [tex3]p^x = 2 [/tex3] , contradição já que p é primo ímpar.
Única solução: [tex3]\boxed{(x,y,p) = (2,2,3)} [/tex3]

Editado pela última vez por Ittalo25 em 07 Abr 2021, 20:12, em um total de 7 vezes.
Ninguém pode ser perfeito, mas todos podem ser melhores. [\Bob Esponja]
Avatar do usuário

Autor do Tópico
Ittalo25
5 - Mestre
Mensagens: 2349
Registrado em: 18 Nov 2013, 22:11
Última visita: 27-03-24
Agradeceu: 299 vezes
Agradeceram: 1401 vezes
Jan 2021 12 22:23

Re: Técnica olímpica - Lifting the exponent (LTE)

Mensagem não lida por Ittalo25 »

Babi123, null

Ninguém pode ser perfeito, mas todos podem ser melhores. [\Bob Esponja]
Responder

Crie uma conta ou entre para participar dessa discussão

Você precisa ser um membro para postar uma resposta

Crie uma nova conta

Ainda não é um membro? Registre-se agora!
Membro pode iniciar seus próprios tópicos e inscrever-se no dos outros para ser notificado sobre atualizações.
É gratuito e leva apenas 1 minuto

Registrar

Entrar

  • Tópicos Semelhantes
    Respostas
    Exibições
    Última mensagem

Voltar para “Demonstrações”